LSAT and Law School Admissions Forum

Get expert LSAT preparation and law school admissions advice from PowerScore Test Preparation.

 Administrator
PowerScore Staff
  • PowerScore Staff
  • Posts: 8916
  • Joined: Feb 02, 2011
|
#85546
Complete Question Explanation

Flaw in the Reasoning. The correct answer choice is (A).

Answer choice (A): This is the correct answer choice.

Answer choice (B):

Answer choice (C):

Answer choice (D):

Answer choice (E):

This explanation is still in progress. Please post any questions below!
User avatar
 mtlwtiener@gmail.com
  • Posts: 2
  • Joined: Dec 30, 2020
|
#87461
I had a hard time with this one because i didnt understand the stimulus and I still dont. What is it saying? and what is it assuming? I figured the conclusion is the sentence after "it follows"but i think that's all i gathered from this stimulus and that its a flaw question.
User avatar
 Ryan Twomey
PowerScore Staff
  • PowerScore Staff
  • Posts: 141
  • Joined: Mar 04, 2021
|
#87517
Hello hello,

So the conclusion of this stimulus is: "It follows that we inhabit a world full of irregular events and in which there are no universal truths."

Essentially the conclusion is just restating the belief of postmodernists without giving any reason as to why they are correct. So the author is siding with postmodernists over the grand theories without any reason, which is a flaw. We would need more evidence as to why the postmodernists theories are correct and why the grand theories are wrong.

Answer choice A essentially states that flaw by saying: "infers that something is the case because it is believed to be the case."

This answer choice is referring to what the author did. The author assumed that we inhabit a world full of irregular events etc, simply because the postmodernists BELIEVE that we do.

I hope this helps and I wish you all the luck in your studies.

Best,
Ryan

Get the most out of your LSAT Prep Plus subscription.

Analyze and track your performance with our Testing and Analytics Package.